ODE Problem: Solution for y'' + (1/x)y' - lambda y = 0 with Boundary Conditions

  • Thread starter member 428835
  • Start date
  • Tags
    Ode
In summary, the conversation discusses an ODE with a variable term and the use of Bessel's equation to solve it. The rescaling of the independent variable is suggested to solve the ODE, and the use of Bessel's functions of first and second kind is mentioned. The question of how to match a constant term to the boundary conditions is brought up.
  • #1
member 428835

Homework Statement


$$y'' + \frac{1}{x}y' - \lambda y = 0$$
where ##x \to \infty \implies y \to 0## and ##x \to 0 \implies y' \to 0##

The Attempt at a Solution


to begin, this was initially a pde, and I've applied separation of variables. to solve this ODE, it seems i cannot assume ##y=e^{rx}## since the ##x^{-1}## term is present. I've thought of a series solution like ##\Sigma_{-\infty}^{\infty} A_n x^n## but the boundary conditions are bringing me to a stop.

any advice would be great! how would you solve this??
 
Physics news on Phys.org
  • #2
joshmccraney said:

Homework Statement


$$y'' + \frac{1}{x}y' - \lambda y = 0$$
where ##x \to \infty \implies y \to 0## and ##x \to 0 \implies y' \to 0##

The Attempt at a Solution


to begin, this was initially a pde, and I've applied separation of variables. to solve this ODE, it seems i cannot assume ##y=e^{rx}## since the ##x^{-1}## term is present. I've thought of a series solution like ##\Sigma_{-\infty}^{\infty} A_n x^n## but the boundary conditions are bringing me to a stop.

any advice would be great! how would you solve this??

Multiplying by [itex]x^2[/itex] gives [tex]
x^2 y'' + xy' - (\lambda x^2)y = 0
[/tex] which if [itex]\lambda = -k^2 < 0[/itex] can be turned into Bessel's equation of order zero by a rescaling of the independent variable; if [itex]\lambda = k^2 > 0[/itex] the ODE can be turned into the modified Bessel's equation of order zero by the same rescaling.

Don't forget the case [itex]\lambda = 0[/itex].
 
  • Like
Likes 1 person
  • #3
pasmith said:
Multiplying by [itex]x^2[/itex] gives [tex]
x^2 y'' + xy' - (\lambda x^2)y = 0
[/tex] which if [itex]\lambda = -k^2 < 0[/itex] can be turned into Bessel's equation of order zero by a rescaling of the independent variable

thanks! could you help me with the rescaling? I'm new to this and not sure how. i was thinking of letting ##x = z / \lambda ^{1/2}## but this doesn't look like it will help me beyond the ##y## term.

thanks so much for the help!
 
  • #4
joshmccraney said:
thanks! could you help me with the rescaling? I'm new to this and not sure how. i was thinking of letting ##x = z / \lambda ^{1/2}## but this doesn't look like it will help me beyond the ##y## term.

thanks so much for the help!
It should help you on all three terms. Don't forget, λ is a constant.

Chet
 
  • Like
Likes 1 person
  • #5
joshmccraney said:
thanks! could you help me with the rescaling? I'm new to this and not sure how. i was thinking of letting ##x = z / \lambda ^{1/2}## but this doesn't look like it will help me beyond the ##y## term.

If [itex]z = kx[/itex], then the chain rule gives [itex]\dfrac{dy}{dx} = k \dfrac{dy}{dz}[/itex]. But [itex]x = z/k[/itex], so [itex]x\dfrac{dy}{dx} = z\dfrac{dy}{dz}[/itex].
 
  • Like
Likes 1 person
  • #6
oh my gosh, of course! ##\frac{dy}{dz} = \frac{dy}{dx} \frac{dx}{dz} \implies \frac{dy}{dz} / \frac{dx}{dz} = \frac{dy}{dx}##

thanks to both of you! (i made a multiplication error). sorry, i should at least triple check before I ask for more help.

thanks!
 
  • #7
i did have another question for you both, if you don't mind?

it seems obvious that the Bessel function of first and second kind on order of zero solve the bessel equation, and it also seems, even with the independent variable change, my boundary conditions stay the same: ##z \to \infty \implies y \to 0## and ##z \to 0 \implies y' \to 0##

my question is, if ##y(z) = a_1 J_0 (z) + a_2 Y_0 (z)## where ##J_0 (z)## and ##Y_0(z)## are bessel functions of first and second kind, respectively, then to satisfy ##z \to 0 \implies y' \to 0## we need to set ##a_2 = 0## from the natural logarithm in ##Y_0##; is this correct?

if so, we now have $$y(z) = a_1J_0(z)$$ where ##z \to \infty \implies y \to 0## governs ##a_1##.

i have two questions left (assuming the above is correct). firstly, the bessel function does converge (by the alternating series test i believe), but what does it converge to? wouldn't i need to know this to fully solve?

and secondly, as i mentioned earlier, Bessels equation arises from a PDE, where i have taken separation of variables. if so, what's the point of solving ##a_1## since we will have more constants from the other ODE (much easier to solve)?

this being said, I'm still real curious on how to match ##a_1## to the b.c.

thanks so much!
 
  • #8
joshmccraney said:
i did have another question for you both, if you don't mind?

it seems obvious that the Bessel function of first and second kind on order of zero solve the bessel equation, and it also seems, even with the independent variable change, my boundary conditions stay the same: ##z \to \infty \implies y \to 0## and ##z \to 0 \implies y' \to 0##

my question is, if ##y(z) = a_1 J_0 (z) + a_2 Y_0 (z)## where ##J_0 (z)## and ##Y_0(z)## are bessel functions of first and second kind, respectively, then to satisfy ##z \to 0 \implies y' \to 0## we need to set ##a_2 = 0## from the natural logarithm in ##Y_0##; is this correct?

if so, we now have $$y(z) = a_1J_0(z)$$ where ##z \to \infty \implies y \to 0## governs ##a_1##.

i have two questions left (assuming the above is correct). firstly, the bessel function does converge (by the alternating series test i believe), but what does it converge to? wouldn't i need to know this to fully solve?

Both [itex]J_0(x)[/itex] and [itex]Y_0(x)[/itex] are oscillatory with the amplitude decaying so that [itex]J_0(x) \to 0[/itex] and [itex]Y_0(x) \to 0[/itex] as [itex]x \to \infty[/itex]. However [itex]Y_0 \to -\infty[/itex] as [itex]x \to 0[/itex], and so cannot be used if x = 0 is in the region you are interested in.

and secondly, as i mentioned earlier, Bessels equation arises from a PDE, where i have taken separation of variables.

Can you post the original PDE and boundary conditions?

if so, what's the point of solving ##a_1## since we will have more constants from the other ODE (much easier to solve)?

You cannot determine [itex]a_1[/itex] from the eigenvalue problem; a constant multiple of an eigenfunction is also an eigenfunction. It is simplest to take [itex]a_1 = 1[/itex].

this being said, I'm still real curious on how to match ##a_1## to the b.c.

It depends on the nature of the boundary conditions. If [itex]0 < x < \infty[/itex] then all values of [itex]\lambda < 0[/itex] are possible and one must use the Hankel transform. This assumes that the eigenvalue problems in the other independent variables don't restrict the possible values of [itex]\lambda[/itex].
 
  • #9
joshmccraney said:
and secondly, as i mentioned earlier, Bessels equation arises from a PDE, where i have taken separation of variables. if so, what's the point of solving ##a_1## since we will have more constants from the other ODE (much easier to solve)?

this being said, I'm still real curious on how to match ##a_1## to the b.c.
You don't solve for a1 separately. You combine it with the constant from the other ODE and then apply the BCs.

Chet
 
  • #10
pasmith said:
Can you post the original PDE and boundary conditions?

sure! it is $$\alpha \frac{\partial f}{\partial x} = \frac{1}{x}\frac{\partial}{\partial u}(x\frac{\partial f}{\partial x}) + \frac{\partial^2 f}{\partial u^2}$$ where ##\alpha## is a constant. this is subject to $$\lim_{x \to \infty}f(x,u) = 0$$ $$\lim_{u \to \infty}f(x,u) = 0$$ $$\lim_{\substack{{u \to 0 \\ x \to 0}}}-4 \pi \frac{1}{u} (x^2+u^2)^{3/2} C \frac{\partial f}{\partial u} = W$$ $$\lim_{x \to 0}\frac{\partial f}{\partial x}= 0$$

and ##C## and ##W## are constants. I've broken this down by separation of variables. now that i can solve the ##x## portion, i think i can get it all. i'll try it tonight (i can't right now) and post what i get!

thanks for both your help!
 
Last edited by a moderator:
  • #11
i should comment here. this boundary condition: $$\lim_{\substack{{u \to 0 \\ x \to 0}}}-4 \pi \frac{1}{u} (x^2+u^2)^{3/2} C \frac{\partial f}{\partial u} = W$$ initially looked like $$\lim_{r \to 0}-4 \pi r^2 C \frac{\partial f}{\partial r}= W$$ such that ##r := \sqrt {u^2 + x^2}##. i changed the boundary condition as shown, which i believe still holds. but, please check me here:
$$\frac{\partial f}{\partial u} = \frac{\partial f}{\partial r} \frac{\partial r}{\partial u}$$ and ##\frac{\partial r}{\partial u} = \frac{u}{r} \implies \frac{\partial f}{\partial r} = \frac{\partial f}{\partial u} \frac{r}{u}##
 
Last edited by a moderator:
  • #12
if either of you are interested, i have formally written the initial problem in latex and turned it into a pdf. it's probably a little too big to post on pf, but if you have an email and are willing to check it out i can forward the .tex and pdf to you.

either way, thanks so much for the help!
 
  • #13
joshmccraney said:
if either of you are interested, i have formally written the initial problem in latex and turned it into a pdf. it's probably a little too big to post on pf, but if you have an email and are willing to check it out i can forward the .tex and pdf to you.

either way, thanks so much for the help!
Sure. I'll send you a PM with my email address.

Chet
 
  • Like
Likes 1 person
  • #14
thanks, i just emailed you!
 

1. What is an ODE problem?

An ODE problem, or ordinary differential equation problem, involves finding the solution to a differential equation that contains one or more independent variables and their derivatives.

2. What does the equation y'' + (1/x)y' - lambda y = 0 represent?

This equation represents a second-order linear homogeneous differential equation. It is commonly used to model physical systems, such as in mechanics and electricity.

3. What does the lambda parameter represent in this equation?

The lambda parameter, also denoted as λ, is a constant value in the equation that can affect the behavior of the solution. It is often referred to as the eigenvalue of the equation.

4. What are the boundary conditions in this ODE problem?

The boundary conditions are conditions that must be satisfied by the solution at the boundaries, or endpoints, of the independent variable. In this equation, the boundary conditions are not specified and may vary depending on the specific problem.

5. How can the solution for this ODE problem be found?

The solution for this ODE problem can be found by using various mathematical techniques, such as separation of variables, variation of parameters, or the method of undetermined coefficients. It may also be possible to solve the equation analytically or numerically using computer software.

Similar threads

  • Calculus and Beyond Homework Help
Replies
11
Views
951
  • Calculus and Beyond Homework Help
Replies
3
Views
307
  • Calculus and Beyond Homework Help
Replies
7
Views
673
  • Calculus and Beyond Homework Help
Replies
2
Views
452
  • Calculus and Beyond Homework Help
Replies
18
Views
1K
  • Calculus and Beyond Homework Help
Replies
2
Views
702
  • Calculus and Beyond Homework Help
Replies
4
Views
1K
  • Calculus and Beyond Homework Help
Replies
1
Views
1K
  • Calculus and Beyond Homework Help
Replies
2
Views
163
  • Calculus and Beyond Homework Help
Replies
3
Views
484
Back
Top